Is this definite integral positive?

The name of the pictureThe name of the pictureThe name of the pictureClash Royale CLAN TAG#URR8PPP











up vote
6
down vote

favorite
2












I have a doubt and I am not able to prove (or disprove):



  • Let $f(x)$ be an odd function with $f(x)>0,,,forall xin (0,+infty)$.

  • Let $g(x)$ be a non-negative function: $g(x)geq 0;forall xin mathbbR.$

  • Also suppose $displaystyle int_-infty^0g(x),dx<int_0^inftyg(x),dx.$

I wonder if one can assure that:



$$int_-infty^inftyf(x),g(x),dx>0.$$



EDIT 1: Has been proved (by Adrian Keister) that my thesis is false.



Now I wonder again if is possible add another hypothesis about $g(x)$ to assure my thesis.



EDIT 2:The problem arrives from here:



enter image description here



blue line is $f(x)= left(e^-fraccosh ^2(u-1)2 -e^-fraccosh ^2(1+u)2 right)$ and orange line is $g(x)=e^-fracu^22 cos ^2left(fracpi (u-1)4 right)$ and the function $f(x)g(x)$ graphic



enter image description here



As we can see in the graph, the integral $int_mathbbRf(x)g(x),dx$ seems to be positive.



We can translate the factor $e^-u^2/2$ from $g(x)$ to $f(x)$ (in this case the third hypothesis is not fulfilled):



enter image description here







share|cite|improve this question


















  • 1




    Can you assume the integral of $g$ is finite? I don't think it would make sense to compare negative with positive infinity.
    – Mefitico
    Aug 29 at 16:44














up vote
6
down vote

favorite
2












I have a doubt and I am not able to prove (or disprove):



  • Let $f(x)$ be an odd function with $f(x)>0,,,forall xin (0,+infty)$.

  • Let $g(x)$ be a non-negative function: $g(x)geq 0;forall xin mathbbR.$

  • Also suppose $displaystyle int_-infty^0g(x),dx<int_0^inftyg(x),dx.$

I wonder if one can assure that:



$$int_-infty^inftyf(x),g(x),dx>0.$$



EDIT 1: Has been proved (by Adrian Keister) that my thesis is false.



Now I wonder again if is possible add another hypothesis about $g(x)$ to assure my thesis.



EDIT 2:The problem arrives from here:



enter image description here



blue line is $f(x)= left(e^-fraccosh ^2(u-1)2 -e^-fraccosh ^2(1+u)2 right)$ and orange line is $g(x)=e^-fracu^22 cos ^2left(fracpi (u-1)4 right)$ and the function $f(x)g(x)$ graphic



enter image description here



As we can see in the graph, the integral $int_mathbbRf(x)g(x),dx$ seems to be positive.



We can translate the factor $e^-u^2/2$ from $g(x)$ to $f(x)$ (in this case the third hypothesis is not fulfilled):



enter image description here







share|cite|improve this question


















  • 1




    Can you assume the integral of $g$ is finite? I don't think it would make sense to compare negative with positive infinity.
    – Mefitico
    Aug 29 at 16:44












up vote
6
down vote

favorite
2









up vote
6
down vote

favorite
2






2





I have a doubt and I am not able to prove (or disprove):



  • Let $f(x)$ be an odd function with $f(x)>0,,,forall xin (0,+infty)$.

  • Let $g(x)$ be a non-negative function: $g(x)geq 0;forall xin mathbbR.$

  • Also suppose $displaystyle int_-infty^0g(x),dx<int_0^inftyg(x),dx.$

I wonder if one can assure that:



$$int_-infty^inftyf(x),g(x),dx>0.$$



EDIT 1: Has been proved (by Adrian Keister) that my thesis is false.



Now I wonder again if is possible add another hypothesis about $g(x)$ to assure my thesis.



EDIT 2:The problem arrives from here:



enter image description here



blue line is $f(x)= left(e^-fraccosh ^2(u-1)2 -e^-fraccosh ^2(1+u)2 right)$ and orange line is $g(x)=e^-fracu^22 cos ^2left(fracpi (u-1)4 right)$ and the function $f(x)g(x)$ graphic



enter image description here



As we can see in the graph, the integral $int_mathbbRf(x)g(x),dx$ seems to be positive.



We can translate the factor $e^-u^2/2$ from $g(x)$ to $f(x)$ (in this case the third hypothesis is not fulfilled):



enter image description here







share|cite|improve this question














I have a doubt and I am not able to prove (or disprove):



  • Let $f(x)$ be an odd function with $f(x)>0,,,forall xin (0,+infty)$.

  • Let $g(x)$ be a non-negative function: $g(x)geq 0;forall xin mathbbR.$

  • Also suppose $displaystyle int_-infty^0g(x),dx<int_0^inftyg(x),dx.$

I wonder if one can assure that:



$$int_-infty^inftyf(x),g(x),dx>0.$$



EDIT 1: Has been proved (by Adrian Keister) that my thesis is false.



Now I wonder again if is possible add another hypothesis about $g(x)$ to assure my thesis.



EDIT 2:The problem arrives from here:



enter image description here



blue line is $f(x)= left(e^-fraccosh ^2(u-1)2 -e^-fraccosh ^2(1+u)2 right)$ and orange line is $g(x)=e^-fracu^22 cos ^2left(fracpi (u-1)4 right)$ and the function $f(x)g(x)$ graphic



enter image description here



As we can see in the graph, the integral $int_mathbbRf(x)g(x),dx$ seems to be positive.



We can translate the factor $e^-u^2/2$ from $g(x)$ to $f(x)$ (in this case the third hypothesis is not fulfilled):



enter image description here









share|cite|improve this question













share|cite|improve this question




share|cite|improve this question








edited Aug 30 at 8:51

























asked Aug 29 at 16:39









popi

9111




9111







  • 1




    Can you assume the integral of $g$ is finite? I don't think it would make sense to compare negative with positive infinity.
    – Mefitico
    Aug 29 at 16:44












  • 1




    Can you assume the integral of $g$ is finite? I don't think it would make sense to compare negative with positive infinity.
    – Mefitico
    Aug 29 at 16:44







1




1




Can you assume the integral of $g$ is finite? I don't think it would make sense to compare negative with positive infinity.
– Mefitico
Aug 29 at 16:44




Can you assume the integral of $g$ is finite? I don't think it would make sense to compare negative with positive infinity.
– Mefitico
Aug 29 at 16:44










2 Answers
2






active

oldest

votes

















up vote
2
down vote



accepted










I should say not. Counterexample:
beginalign*
f(x)&=left
beginarrayll
-1/(10x^2), ; &xin(-infty,-1) \
-1, ; &xin[-1,0) \
0, ; &x=0 \
1, ; &xin(0,1] \
1/(10x^2), ; &xin(1,infty)
endarrayright, \
g(x)&=leftbeginarrayll
0,;&xin(-infty,-1)cup(0,1)cup(2,infty) \
1/2, &xin[-1,0] \
1, &xin[1,2] ; endarrayright.
endalign*



If you replace the $displaystyle int_-infty^0g(x),dx<int_0^inftyg(x),dx$ condition with the (stronger) condition $g(-x)<g(x);forall,x>0,$ then you have the following:
beginalign*
int_-infty^inftyf(x),g(x),dx&=
int_-infty^0f(x),g(x),dx+int_0^inftyf(x),g(x),dx \
&=-int_infty^0 f(-x),g(-x),dx+int_0^inftyf(x),g(x),dx \
&=int_0^infty f(-x),g(-x),dx+int_0^inftyf(x),g(x),dx \
&=-int_0^infty f(x),g(-x),dx+int_0^inftyf(x),g(x),dx \
&=int_0^infty f(x),(-g(-x)),dx+int_0^inftyf(x),g(x),dx. \
endalign*
Now, if you have $g(-x)<g(x);forall,x>0,$ it follows that $-g(-x)>-g(x),$ whence you get
beginalign*
int_-infty^inftyf(x),g(x),dx&=
int_0^infty f(x),(-g(-x)),dx+int_0^inftyf(x),g(x),dx \
&>-int_0^infty f(x),g(x),dx+int_0^inftyf(x),g(x),dx \
&=0.
endalign*



See tomasz's answer for a slightly less strict condition on $g(x)$.






share|cite|improve this answer






















  • @popi: Yeah, I saw that. Not sure what you could do in addition. If you say $g$ is even, your integral condition can't hold (you'd actually get the integral in question to be zero).
    – Adrian Keister
    Aug 29 at 17:39










  • Something about monotony, absolute or relative extremes, etc ... of $ g (x) $.
    – popi
    Aug 29 at 17:48










  • What about $g(-x)<g(x);forall,x>0?$
    – Adrian Keister
    Aug 29 at 17:54











  • See tomasz's answer for a slightly less restrictive condition on $g(x)$ that works. I think my proof could be adapted slightly for tomasz's condition.
    – Adrian Keister
    Aug 29 at 18:10










  • Your way of positioning commas and periods in your piecewise definitions seems very strange. I would write $$ f(x) = begincases -1/(10x^2), ; &xin(-infty,-1), \ -1, ; &xin[-1,0), \ 0, ; &x=0, \ 1, ; &xin(0,1], \ 1/(10x^2), ; &xin(1,infty). endcases $$
    – Michael Hardy
    Aug 29 at 18:19

















up vote
2
down vote













If for almost all positive $x$ you have $g(x)geq g(-x)$ and that there is a non-null set of $x$ such that $g(x)neq g(-x)$, then this integral will always be positive.



No other hypothesis on $g$ will suffice (for all $f$). If $g(x)=g(-x)$ almost everywhere, then the integral is clearly zero. Otherwise, suppose $Asubseteq [0,infty)$ is a set of positive measure such that for $xin A$ we have $g(x)< g(-x)$. We may assume without loss of generality that for all $xin A$ we have $g(-x)>g(x)+1/n$ for some positive integer $n$.



Now, take for $f$ an odd function such that $f(x)=2n/lvert Arvert$ for $xin A$ and $f(x)<left(int_-infty^infty lvert g(t)rvert,mathrmdtright)^-1$ for positive $xnotin A$.



Then $int_Acup -A f(t)g(t),mathrmdt<-2$ and $leftlvert int_(Acup -A)^c f(t)g(t),mathrmdtrightrvertleq 1$.






share|cite|improve this answer




















    Your Answer




    StackExchange.ifUsing("editor", function ()
    return StackExchange.using("mathjaxEditing", function ()
    StackExchange.MarkdownEditor.creationCallbacks.add(function (editor, postfix)
    StackExchange.mathjaxEditing.prepareWmdForMathJax(editor, postfix, [["$", "$"], ["\\(","\\)"]]);
    );
    );
    , "mathjax-editing");

    StackExchange.ready(function()
    var channelOptions =
    tags: "".split(" "),
    id: "69"
    ;
    initTagRenderer("".split(" "), "".split(" "), channelOptions);

    StackExchange.using("externalEditor", function()
    // Have to fire editor after snippets, if snippets enabled
    if (StackExchange.settings.snippets.snippetsEnabled)
    StackExchange.using("snippets", function()
    createEditor();
    );

    else
    createEditor();

    );

    function createEditor()
    StackExchange.prepareEditor(
    heartbeatType: 'answer',
    convertImagesToLinks: true,
    noModals: false,
    showLowRepImageUploadWarning: true,
    reputationToPostImages: 10,
    bindNavPrevention: true,
    postfix: "",
    noCode: true, onDemand: true,
    discardSelector: ".discard-answer"
    ,immediatelyShowMarkdownHelp:true
    );



    );













     

    draft saved


    draft discarded


















    StackExchange.ready(
    function ()
    StackExchange.openid.initPostLogin('.new-post-login', 'https%3a%2f%2fmath.stackexchange.com%2fquestions%2f2898557%2fis-this-definite-integral-positive%23new-answer', 'question_page');

    );

    Post as a guest






























    2 Answers
    2






    active

    oldest

    votes








    2 Answers
    2






    active

    oldest

    votes









    active

    oldest

    votes






    active

    oldest

    votes








    up vote
    2
    down vote



    accepted










    I should say not. Counterexample:
    beginalign*
    f(x)&=left
    beginarrayll
    -1/(10x^2), ; &xin(-infty,-1) \
    -1, ; &xin[-1,0) \
    0, ; &x=0 \
    1, ; &xin(0,1] \
    1/(10x^2), ; &xin(1,infty)
    endarrayright, \
    g(x)&=leftbeginarrayll
    0,;&xin(-infty,-1)cup(0,1)cup(2,infty) \
    1/2, &xin[-1,0] \
    1, &xin[1,2] ; endarrayright.
    endalign*



    If you replace the $displaystyle int_-infty^0g(x),dx<int_0^inftyg(x),dx$ condition with the (stronger) condition $g(-x)<g(x);forall,x>0,$ then you have the following:
    beginalign*
    int_-infty^inftyf(x),g(x),dx&=
    int_-infty^0f(x),g(x),dx+int_0^inftyf(x),g(x),dx \
    &=-int_infty^0 f(-x),g(-x),dx+int_0^inftyf(x),g(x),dx \
    &=int_0^infty f(-x),g(-x),dx+int_0^inftyf(x),g(x),dx \
    &=-int_0^infty f(x),g(-x),dx+int_0^inftyf(x),g(x),dx \
    &=int_0^infty f(x),(-g(-x)),dx+int_0^inftyf(x),g(x),dx. \
    endalign*
    Now, if you have $g(-x)<g(x);forall,x>0,$ it follows that $-g(-x)>-g(x),$ whence you get
    beginalign*
    int_-infty^inftyf(x),g(x),dx&=
    int_0^infty f(x),(-g(-x)),dx+int_0^inftyf(x),g(x),dx \
    &>-int_0^infty f(x),g(x),dx+int_0^inftyf(x),g(x),dx \
    &=0.
    endalign*



    See tomasz's answer for a slightly less strict condition on $g(x)$.






    share|cite|improve this answer






















    • @popi: Yeah, I saw that. Not sure what you could do in addition. If you say $g$ is even, your integral condition can't hold (you'd actually get the integral in question to be zero).
      – Adrian Keister
      Aug 29 at 17:39










    • Something about monotony, absolute or relative extremes, etc ... of $ g (x) $.
      – popi
      Aug 29 at 17:48










    • What about $g(-x)<g(x);forall,x>0?$
      – Adrian Keister
      Aug 29 at 17:54











    • See tomasz's answer for a slightly less restrictive condition on $g(x)$ that works. I think my proof could be adapted slightly for tomasz's condition.
      – Adrian Keister
      Aug 29 at 18:10










    • Your way of positioning commas and periods in your piecewise definitions seems very strange. I would write $$ f(x) = begincases -1/(10x^2), ; &xin(-infty,-1), \ -1, ; &xin[-1,0), \ 0, ; &x=0, \ 1, ; &xin(0,1], \ 1/(10x^2), ; &xin(1,infty). endcases $$
      – Michael Hardy
      Aug 29 at 18:19














    up vote
    2
    down vote



    accepted










    I should say not. Counterexample:
    beginalign*
    f(x)&=left
    beginarrayll
    -1/(10x^2), ; &xin(-infty,-1) \
    -1, ; &xin[-1,0) \
    0, ; &x=0 \
    1, ; &xin(0,1] \
    1/(10x^2), ; &xin(1,infty)
    endarrayright, \
    g(x)&=leftbeginarrayll
    0,;&xin(-infty,-1)cup(0,1)cup(2,infty) \
    1/2, &xin[-1,0] \
    1, &xin[1,2] ; endarrayright.
    endalign*



    If you replace the $displaystyle int_-infty^0g(x),dx<int_0^inftyg(x),dx$ condition with the (stronger) condition $g(-x)<g(x);forall,x>0,$ then you have the following:
    beginalign*
    int_-infty^inftyf(x),g(x),dx&=
    int_-infty^0f(x),g(x),dx+int_0^inftyf(x),g(x),dx \
    &=-int_infty^0 f(-x),g(-x),dx+int_0^inftyf(x),g(x),dx \
    &=int_0^infty f(-x),g(-x),dx+int_0^inftyf(x),g(x),dx \
    &=-int_0^infty f(x),g(-x),dx+int_0^inftyf(x),g(x),dx \
    &=int_0^infty f(x),(-g(-x)),dx+int_0^inftyf(x),g(x),dx. \
    endalign*
    Now, if you have $g(-x)<g(x);forall,x>0,$ it follows that $-g(-x)>-g(x),$ whence you get
    beginalign*
    int_-infty^inftyf(x),g(x),dx&=
    int_0^infty f(x),(-g(-x)),dx+int_0^inftyf(x),g(x),dx \
    &>-int_0^infty f(x),g(x),dx+int_0^inftyf(x),g(x),dx \
    &=0.
    endalign*



    See tomasz's answer for a slightly less strict condition on $g(x)$.






    share|cite|improve this answer






















    • @popi: Yeah, I saw that. Not sure what you could do in addition. If you say $g$ is even, your integral condition can't hold (you'd actually get the integral in question to be zero).
      – Adrian Keister
      Aug 29 at 17:39










    • Something about monotony, absolute or relative extremes, etc ... of $ g (x) $.
      – popi
      Aug 29 at 17:48










    • What about $g(-x)<g(x);forall,x>0?$
      – Adrian Keister
      Aug 29 at 17:54











    • See tomasz's answer for a slightly less restrictive condition on $g(x)$ that works. I think my proof could be adapted slightly for tomasz's condition.
      – Adrian Keister
      Aug 29 at 18:10










    • Your way of positioning commas and periods in your piecewise definitions seems very strange. I would write $$ f(x) = begincases -1/(10x^2), ; &xin(-infty,-1), \ -1, ; &xin[-1,0), \ 0, ; &x=0, \ 1, ; &xin(0,1], \ 1/(10x^2), ; &xin(1,infty). endcases $$
      – Michael Hardy
      Aug 29 at 18:19












    up vote
    2
    down vote



    accepted







    up vote
    2
    down vote



    accepted






    I should say not. Counterexample:
    beginalign*
    f(x)&=left
    beginarrayll
    -1/(10x^2), ; &xin(-infty,-1) \
    -1, ; &xin[-1,0) \
    0, ; &x=0 \
    1, ; &xin(0,1] \
    1/(10x^2), ; &xin(1,infty)
    endarrayright, \
    g(x)&=leftbeginarrayll
    0,;&xin(-infty,-1)cup(0,1)cup(2,infty) \
    1/2, &xin[-1,0] \
    1, &xin[1,2] ; endarrayright.
    endalign*



    If you replace the $displaystyle int_-infty^0g(x),dx<int_0^inftyg(x),dx$ condition with the (stronger) condition $g(-x)<g(x);forall,x>0,$ then you have the following:
    beginalign*
    int_-infty^inftyf(x),g(x),dx&=
    int_-infty^0f(x),g(x),dx+int_0^inftyf(x),g(x),dx \
    &=-int_infty^0 f(-x),g(-x),dx+int_0^inftyf(x),g(x),dx \
    &=int_0^infty f(-x),g(-x),dx+int_0^inftyf(x),g(x),dx \
    &=-int_0^infty f(x),g(-x),dx+int_0^inftyf(x),g(x),dx \
    &=int_0^infty f(x),(-g(-x)),dx+int_0^inftyf(x),g(x),dx. \
    endalign*
    Now, if you have $g(-x)<g(x);forall,x>0,$ it follows that $-g(-x)>-g(x),$ whence you get
    beginalign*
    int_-infty^inftyf(x),g(x),dx&=
    int_0^infty f(x),(-g(-x)),dx+int_0^inftyf(x),g(x),dx \
    &>-int_0^infty f(x),g(x),dx+int_0^inftyf(x),g(x),dx \
    &=0.
    endalign*



    See tomasz's answer for a slightly less strict condition on $g(x)$.






    share|cite|improve this answer














    I should say not. Counterexample:
    beginalign*
    f(x)&=left
    beginarrayll
    -1/(10x^2), ; &xin(-infty,-1) \
    -1, ; &xin[-1,0) \
    0, ; &x=0 \
    1, ; &xin(0,1] \
    1/(10x^2), ; &xin(1,infty)
    endarrayright, \
    g(x)&=leftbeginarrayll
    0,;&xin(-infty,-1)cup(0,1)cup(2,infty) \
    1/2, &xin[-1,0] \
    1, &xin[1,2] ; endarrayright.
    endalign*



    If you replace the $displaystyle int_-infty^0g(x),dx<int_0^inftyg(x),dx$ condition with the (stronger) condition $g(-x)<g(x);forall,x>0,$ then you have the following:
    beginalign*
    int_-infty^inftyf(x),g(x),dx&=
    int_-infty^0f(x),g(x),dx+int_0^inftyf(x),g(x),dx \
    &=-int_infty^0 f(-x),g(-x),dx+int_0^inftyf(x),g(x),dx \
    &=int_0^infty f(-x),g(-x),dx+int_0^inftyf(x),g(x),dx \
    &=-int_0^infty f(x),g(-x),dx+int_0^inftyf(x),g(x),dx \
    &=int_0^infty f(x),(-g(-x)),dx+int_0^inftyf(x),g(x),dx. \
    endalign*
    Now, if you have $g(-x)<g(x);forall,x>0,$ it follows that $-g(-x)>-g(x),$ whence you get
    beginalign*
    int_-infty^inftyf(x),g(x),dx&=
    int_0^infty f(x),(-g(-x)),dx+int_0^inftyf(x),g(x),dx \
    &>-int_0^infty f(x),g(x),dx+int_0^inftyf(x),g(x),dx \
    &=0.
    endalign*



    See tomasz's answer for a slightly less strict condition on $g(x)$.







    share|cite|improve this answer














    share|cite|improve this answer



    share|cite|improve this answer








    edited Aug 29 at 18:43

























    answered Aug 29 at 17:16









    Adrian Keister

    4,04541633




    4,04541633











    • @popi: Yeah, I saw that. Not sure what you could do in addition. If you say $g$ is even, your integral condition can't hold (you'd actually get the integral in question to be zero).
      – Adrian Keister
      Aug 29 at 17:39










    • Something about monotony, absolute or relative extremes, etc ... of $ g (x) $.
      – popi
      Aug 29 at 17:48










    • What about $g(-x)<g(x);forall,x>0?$
      – Adrian Keister
      Aug 29 at 17:54











    • See tomasz's answer for a slightly less restrictive condition on $g(x)$ that works. I think my proof could be adapted slightly for tomasz's condition.
      – Adrian Keister
      Aug 29 at 18:10










    • Your way of positioning commas and periods in your piecewise definitions seems very strange. I would write $$ f(x) = begincases -1/(10x^2), ; &xin(-infty,-1), \ -1, ; &xin[-1,0), \ 0, ; &x=0, \ 1, ; &xin(0,1], \ 1/(10x^2), ; &xin(1,infty). endcases $$
      – Michael Hardy
      Aug 29 at 18:19
















    • @popi: Yeah, I saw that. Not sure what you could do in addition. If you say $g$ is even, your integral condition can't hold (you'd actually get the integral in question to be zero).
      – Adrian Keister
      Aug 29 at 17:39










    • Something about monotony, absolute or relative extremes, etc ... of $ g (x) $.
      – popi
      Aug 29 at 17:48










    • What about $g(-x)<g(x);forall,x>0?$
      – Adrian Keister
      Aug 29 at 17:54











    • See tomasz's answer for a slightly less restrictive condition on $g(x)$ that works. I think my proof could be adapted slightly for tomasz's condition.
      – Adrian Keister
      Aug 29 at 18:10










    • Your way of positioning commas and periods in your piecewise definitions seems very strange. I would write $$ f(x) = begincases -1/(10x^2), ; &xin(-infty,-1), \ -1, ; &xin[-1,0), \ 0, ; &x=0, \ 1, ; &xin(0,1], \ 1/(10x^2), ; &xin(1,infty). endcases $$
      – Michael Hardy
      Aug 29 at 18:19















    @popi: Yeah, I saw that. Not sure what you could do in addition. If you say $g$ is even, your integral condition can't hold (you'd actually get the integral in question to be zero).
    – Adrian Keister
    Aug 29 at 17:39




    @popi: Yeah, I saw that. Not sure what you could do in addition. If you say $g$ is even, your integral condition can't hold (you'd actually get the integral in question to be zero).
    – Adrian Keister
    Aug 29 at 17:39












    Something about monotony, absolute or relative extremes, etc ... of $ g (x) $.
    – popi
    Aug 29 at 17:48




    Something about monotony, absolute or relative extremes, etc ... of $ g (x) $.
    – popi
    Aug 29 at 17:48












    What about $g(-x)<g(x);forall,x>0?$
    – Adrian Keister
    Aug 29 at 17:54





    What about $g(-x)<g(x);forall,x>0?$
    – Adrian Keister
    Aug 29 at 17:54













    See tomasz's answer for a slightly less restrictive condition on $g(x)$ that works. I think my proof could be adapted slightly for tomasz's condition.
    – Adrian Keister
    Aug 29 at 18:10




    See tomasz's answer for a slightly less restrictive condition on $g(x)$ that works. I think my proof could be adapted slightly for tomasz's condition.
    – Adrian Keister
    Aug 29 at 18:10












    Your way of positioning commas and periods in your piecewise definitions seems very strange. I would write $$ f(x) = begincases -1/(10x^2), ; &xin(-infty,-1), \ -1, ; &xin[-1,0), \ 0, ; &x=0, \ 1, ; &xin(0,1], \ 1/(10x^2), ; &xin(1,infty). endcases $$
    – Michael Hardy
    Aug 29 at 18:19




    Your way of positioning commas and periods in your piecewise definitions seems very strange. I would write $$ f(x) = begincases -1/(10x^2), ; &xin(-infty,-1), \ -1, ; &xin[-1,0), \ 0, ; &x=0, \ 1, ; &xin(0,1], \ 1/(10x^2), ; &xin(1,infty). endcases $$
    – Michael Hardy
    Aug 29 at 18:19










    up vote
    2
    down vote













    If for almost all positive $x$ you have $g(x)geq g(-x)$ and that there is a non-null set of $x$ such that $g(x)neq g(-x)$, then this integral will always be positive.



    No other hypothesis on $g$ will suffice (for all $f$). If $g(x)=g(-x)$ almost everywhere, then the integral is clearly zero. Otherwise, suppose $Asubseteq [0,infty)$ is a set of positive measure such that for $xin A$ we have $g(x)< g(-x)$. We may assume without loss of generality that for all $xin A$ we have $g(-x)>g(x)+1/n$ for some positive integer $n$.



    Now, take for $f$ an odd function such that $f(x)=2n/lvert Arvert$ for $xin A$ and $f(x)<left(int_-infty^infty lvert g(t)rvert,mathrmdtright)^-1$ for positive $xnotin A$.



    Then $int_Acup -A f(t)g(t),mathrmdt<-2$ and $leftlvert int_(Acup -A)^c f(t)g(t),mathrmdtrightrvertleq 1$.






    share|cite|improve this answer
























      up vote
      2
      down vote













      If for almost all positive $x$ you have $g(x)geq g(-x)$ and that there is a non-null set of $x$ such that $g(x)neq g(-x)$, then this integral will always be positive.



      No other hypothesis on $g$ will suffice (for all $f$). If $g(x)=g(-x)$ almost everywhere, then the integral is clearly zero. Otherwise, suppose $Asubseteq [0,infty)$ is a set of positive measure such that for $xin A$ we have $g(x)< g(-x)$. We may assume without loss of generality that for all $xin A$ we have $g(-x)>g(x)+1/n$ for some positive integer $n$.



      Now, take for $f$ an odd function such that $f(x)=2n/lvert Arvert$ for $xin A$ and $f(x)<left(int_-infty^infty lvert g(t)rvert,mathrmdtright)^-1$ for positive $xnotin A$.



      Then $int_Acup -A f(t)g(t),mathrmdt<-2$ and $leftlvert int_(Acup -A)^c f(t)g(t),mathrmdtrightrvertleq 1$.






      share|cite|improve this answer






















        up vote
        2
        down vote










        up vote
        2
        down vote









        If for almost all positive $x$ you have $g(x)geq g(-x)$ and that there is a non-null set of $x$ such that $g(x)neq g(-x)$, then this integral will always be positive.



        No other hypothesis on $g$ will suffice (for all $f$). If $g(x)=g(-x)$ almost everywhere, then the integral is clearly zero. Otherwise, suppose $Asubseteq [0,infty)$ is a set of positive measure such that for $xin A$ we have $g(x)< g(-x)$. We may assume without loss of generality that for all $xin A$ we have $g(-x)>g(x)+1/n$ for some positive integer $n$.



        Now, take for $f$ an odd function such that $f(x)=2n/lvert Arvert$ for $xin A$ and $f(x)<left(int_-infty^infty lvert g(t)rvert,mathrmdtright)^-1$ for positive $xnotin A$.



        Then $int_Acup -A f(t)g(t),mathrmdt<-2$ and $leftlvert int_(Acup -A)^c f(t)g(t),mathrmdtrightrvertleq 1$.






        share|cite|improve this answer












        If for almost all positive $x$ you have $g(x)geq g(-x)$ and that there is a non-null set of $x$ such that $g(x)neq g(-x)$, then this integral will always be positive.



        No other hypothesis on $g$ will suffice (for all $f$). If $g(x)=g(-x)$ almost everywhere, then the integral is clearly zero. Otherwise, suppose $Asubseteq [0,infty)$ is a set of positive measure such that for $xin A$ we have $g(x)< g(-x)$. We may assume without loss of generality that for all $xin A$ we have $g(-x)>g(x)+1/n$ for some positive integer $n$.



        Now, take for $f$ an odd function such that $f(x)=2n/lvert Arvert$ for $xin A$ and $f(x)<left(int_-infty^infty lvert g(t)rvert,mathrmdtright)^-1$ for positive $xnotin A$.



        Then $int_Acup -A f(t)g(t),mathrmdt<-2$ and $leftlvert int_(Acup -A)^c f(t)g(t),mathrmdtrightrvertleq 1$.







        share|cite|improve this answer












        share|cite|improve this answer



        share|cite|improve this answer










        answered Aug 29 at 18:05









        tomasz

        22.9k23077




        22.9k23077



























             

            draft saved


            draft discarded















































             


            draft saved


            draft discarded














            StackExchange.ready(
            function ()
            StackExchange.openid.initPostLogin('.new-post-login', 'https%3a%2f%2fmath.stackexchange.com%2fquestions%2f2898557%2fis-this-definite-integral-positive%23new-answer', 'question_page');

            );

            Post as a guest













































































            Comments

            Popular posts from this blog

            Long meetings (6-7 hours a day): Being “babysat” by supervisor

            Is the Concept of Multiple Fantasy Races Scientifically Flawed? [closed]

            Confectionery